La ricerca ha trovato 109 risultati

da g(n)
20 mar 2009, 21:13
Forum: Gara a squadre
Argomento: Risultati 2009
Risposte: 87
Visite : 43901

Gara di Pordenone, noi del Marinelli di Udine abbiamo fatto 2014 punti, dando un distacco di un migliaio di punti ai secondi :D Uè, non tirartela tanto... :? ...in fondo non li abbiamo neanche doppiati i secondi (per un centinaio di punti) :twisted: :twisted: :twisted: muahahahahaha Per inciso, pas...
da g(n)
20 feb 2009, 18:24
Forum: Teoria dei Numeri
Argomento: Visto che siamo in vena di diofantee...
Risposte: 9
Visite : 4260

C'è come (quasi) sempre anche il metodo bovino: la considero come un'equazione di 2° grado in x e pongo il delta uguale ad un quadrato perfetto
da g(n)
22 gen 2009, 17:11
Forum: Teoria dei Numeri
Argomento: Diofantea (titolo insolito...)
Risposte: 4
Visite : 2283

Altrimenti:
casi 2 e 3 a mano; poi, per $ p\geq 5 $ primo, si ha che $ p^2\equiv 1 \pmod 6 $
Quindi deve risultare
$ n-2\equiv n^6\pmod 6 $

impossibile (verificate :) )
da g(n)
17 gen 2009, 00:02
Forum: Olimpiadi della matematica
Argomento: Winter Camp 2009
Risposte: 89
Visite : 38700

Evvai!! :D

Condoglianze a ghilu se ha intenzione di fare un lavoro completo leggendosi tutte le soluzioni...
da g(n)
16 gen 2009, 23:37
Forum: Algebra
Argomento: Ingrandendo i fattori...
Risposte: 8
Visite : 3965

Volendo si può svolgere il prodotto, vedere che saltano fuori 2^n addendi, applicare AM-GM e, per simmetria, ogni $ a_i $ comparirà lo stesso numero di volte nella GM, cosicchè GM=1, da cui la tesi.

@Jacobi: :o ottima soluzione, non ci avevo pensato :D
da g(n)
16 gen 2009, 17:25
Forum: Algebra
Argomento: Ingrandendo i fattori...
Risposte: 8
Visite : 3965

Sì, giusto :D

Volevo proporre una mini generalizzazione:

$ (m+a_1)\cdot \ldots \cdot(m+a_n)\geq (m+1)^n $

ma dovrebbe venire automatica a questo punto.
da g(n)
16 gen 2009, 10:23
Forum: Algebra
Argomento: Ingrandendo i fattori...
Risposte: 8
Visite : 3965

Ingrandendo i fattori...

Non so se sia già stato postato, comunque lo metto lo stesso. Dati $ a_1,\ldots,a_n $ reali positivi tali che $ a_1\cdot \ldots\cdot a_n=1 $, dimostrare che

$ (1+a_1)\cdot \ldots\cdot (1+a_n)\geq 2^n $

Buon lavoro :D
da g(n)
09 gen 2009, 18:59
Forum: Teoria dei Numeri
Argomento: Interi positivi come somme d'interi positivi consecutivi-own
Risposte: 19
Visite : 7238

Penso che fosse sottinteso :) Altrimenti, come hai notato tu, il problema era piuttosto banale!
da g(n)
09 gen 2009, 18:55
Forum: Teoria dei Numeri
Argomento: residui quadratici modulo 2^n
Risposte: 8
Visite : 3709

La mia soluzione è molto simile a quella di darkcrystal, ma leggermente diversa, quindi la posto (anche perchè mi ha bruciato sul tempo, l'avevo risolto ma per mancanza di tempo non l'ho postato :x :x ) Voglio dimostrare che gli insiemi \{1^2,3^2,...,(2^{n-2}-1)^2\} e \{1,9,...,2^{n}-7\} coincidono ...
da g(n)
08 gen 2009, 17:27
Forum: Teoria dei Numeri
Argomento: doppi primi
Risposte: 8
Visite : 3085

Scusa, magari ho letto di fretta, ma forse intendevi
$ \displaystyle \sum_{k=1}^{p-1}\binom pk \binom q{p-k} $
o sbaglio?

Comunque la divisibilità rimane lo stesso, quindi non cambia niente.

Bella soluzione comunque :wink: :D
da g(n)
06 gen 2009, 01:35
Forum: Teoria dei Numeri
Argomento: cosa lo divide?
Risposte: 9
Visite : 4022

Sì sì, è come dici tu. In generale quando si ha una "torre" di potenze, si parte da quella più interna, cioè quella più "alta". Per esempio
$ 2^{3^{4^{5}}}=2^{(3^{(4^5)})} $
e se ci pensi le parentesi sono come le scrivi in LaTeX
da g(n)
02 gen 2009, 18:03
Forum: Matematica non elementare
Argomento: È elementare? (Own)
Risposte: 4
Visite : 3228

Solito ritardo da LaTeX (e da lentezza del computer) :)

PS in effetti serve che siano positivi...
da g(n)
02 gen 2009, 17:59
Forum: Matematica non elementare
Argomento: È elementare? (Own)
Risposte: 4
Visite : 3228

Mi sembra semplice...anche dopo l'avvertimento :D :D Supponiamo wlog che x sia il massimo. Allora x^2+y^2+z^2\leq 3x^2 con uguaglianza sse x=y=z , e quindi \displaystyle\frac{\sqrt{x^2+y^2+z^2}}{max\{x,y,z\}}= \frac{\sqrt{x^2+y^2+z^2}}{x} \leq \frac{\sqrt{3x^2}}{x} =\sqrt 3 e l'uguaglianza vale appu...
da g(n)
02 gen 2009, 17:28
Forum: Fisica
Argomento: Minimizzare AP + BP + CP
Risposte: 9
Visite : 8302

:o :o :o Fantastico :D

Pig, l'hai pensato tu?
da g(n)
30 dic 2008, 01:12
Forum: Algebra
Argomento: la + piccola differenza
Risposte: 10
Visite : 4479

Ok, era solo per chiedere...non preoccuparti, non posto la soluzione...non vorrei commettere un infanticidio :)